You are on page 1of 180
SCHEMA Ty SCT riers M Celie [Cy co oC) LUE is Complete Answers and ela iiseexe aqua Yim Wai-leung Contents Part 4 Toa Toate ru Cete a au) Unit 1 Electrostatics Unit 2 Circuits and Domestic Electricity | Unit 3 Circuits and Domestic Electricity I! Unit 4 Electromagnetism | Unit 5 Electromagnetism II Complete Answers and Marking Schemes [Ey “Presents that the material is not included in the Physics part ‘of Combined Science represents the extension part of the curriculum eC Lh) Name: Class : ae Date: 1 rn 2 5 3 16 Hi 7 3 18 a @ 7 20 3 2 . 2 0 3 u 2 R 25 n Toad 78 @ Examination Analysis Past Examination Paper Analysis Examination Trend ... © Useful Formulae © Examination Scoring Techniques . © Common Mistakes .... © Revision Test Multiple-choice Questions . Short Question: Structured Questions Essays. Telia Menai} rear & Part 4 Blectricty and Magnetism o 4 @yekamination analysis ££ Past Examination Paper Analysis (a) Multiple-choice Questions HKCEE HKAL Topic Year | Paper | Question No. | Year | Paper | Question No. 2003 | 44 2001 | mA 20 2004 | 0 27 2004 | IA 15,24 Electrostatic phenomena ++ 2005 | 7 2006 | 2 2001 | 28 2000 | MA 21 2003 | A 31 2003 | IA 23,26 Coulomb’ law 2007 | 19 2005 | WA 12, 34 2006 | TIA 10,11 Different charging methods 2006 | IA 2 1999 | A 21 2000 | IA 2 Electric field 2004 | lA 18 2009 | IA 24,33 2003 | 6 2002 | iA 20 2003 | IIA 24 Electric potential 2004 | IA 16 2oo8 | WA | 13,14,34 (b) Short Questions / Structured Questions Year | Paper | Question No.| Marks Content 2002 | AL 3 10 | Electric potential of a charged sphere. Paper IA 2003 | AL 6 16 | Electric potential between an E.H.T. charged sphere and Paper 1B a flame probe. 2006 | AL 4 13. | Adeam of electrons enters a region of electric field Paper IA between two parallel plates. © 2010 Times Publishing (Hong Kong) Limited -2- Physies Success Key for HKDSEE 4 Electrostatic (c) Essays Year | Paper | Question No.| Marks Content 2001 | AL 3a) 4 | Define electric field intensity and electric potential at a Paper IIB point in an electric field. 2006 | AL 3a) 5 | Describe the meaning of electric field and describe the Paper IIB electric field pattern between two parallel plates and that between a point electrode and a straight electrode. Examination Trend (a) Interpret charging in terms of electron transfer, (b) Apply Coulomb’ law to solve problems involving interaction force between point charges (©) Describe an electric field pattern of an electric field by using field lines (d) Study electric field lines and the moving path ofa charge in an electric field. (©) Solve problems involving electric field strength and electric potential around a point charge (O_ Solve problems involving electric field strength and electric potential around two point charges, (g) Describe an electric field between two parallel plates and solve related problems, e.g. a charge moving, between the parallel plates. Useful Formulae A$ AA_€pNANA—aa_{_a_—$______ Useful term Formula 1, Coulomb force (force between two point charges) 7 : (F = Coulomb interaction force, Oy and Q; eparation between two point charges) © 2010 Times Publishing (Hong Kang) Limited -3- hysies Success Key for HKDSEE 4 on 4 Electricity and Magnetism Useful term Formula 2. Electric field due to a point | @® For positive point charges charge e (ii) For negative point charges e pa ne, (E = electric field, Q = charge, = distance from a point to a point charge) 3. Electric field between parallel plates (numerically) ence : oe a (V = potential difference, d = separation between two plates) (© 2010 Times Publishing (Hong Kong) Limited Physics Suceess Key for HKDSEE 4 9 pate] ters iy Useful term Formula 4. Electric potential due to a | (i) For positive point charges int char : . wv rim Gi) For negative point charges viv rm 1Q Ve one, 5. Work done on a charge W= OAV (W = work done, AV = change of potential) 6. Relationship between V and E (AV = change of potential, Ar = change of distance) Physies Success Key for HKDSEE 4 oe 4 electricity and Magnetisne & Examination Scoring Techniques —————. 1. Charging by Induction ‘Conductors can be charged by induction and the steps are as follows: Step 1: Bring a positively charged rod neara Step 2: Earth the sphere, electrons will then conducting sphere, negative charges in flow from the Earth to the sphere so the sphere will be attracted towards the that the sphere has net negative charges. i L ae Step 3: Remove the earthing, the negative Step 4: Remove the rod, the sphere then charges will remain in the sphere. becomes negatively charged Remarks: Ifa negatively charged rod is used and the same steps are fotlowed, then the conelucting sphere will be positively charged. 2. Extracting Information from Electric Field Lines {© 2010 Times Publishing (Hong Kong) Limited -6- Physies Success Key for HKDSEE 4 oe): Stee The above figure shows a pattern of electric field lines in which A, Band C are points marked on one of the field lines. The following information can be extracted from the figute (The electric potential V at different points are of the order V, > Vp > Ve. Gii) The electric field strength E at different points are of the order E, < Ey < Ec (Gii) Ifa positive charge is released! from rest at A, it will accelerate towards C along the field line, The electric force acting on the positive charge directs to the right along the tangent to the field line while the positive charge is moving on the field line as shown in the following Figure. the electric foros dveats along the tangent tothe field tine positive charge (iv) Positive work has to be done on a positive charge if it is moved from B to A or from C to A. Extracting Information from the Moving Path of a Charged Particle in an Electric Field = The solid lines in the above figure represent part of an electric field due to a fixed point charge Q (not shown In the figure). A charged particle q, subjected only to electric force in the field, travels along the dotted curve as shown. The following information can be extracted from the figure: (® The electric force between the two charges must be attractive. Ifthe electric force is repulsive, the path of q should be as follows: (a) Forq starting from X Va Hence, charges flow from sphere A to B until they have the same surface potential, Let Q be the charges lowing from A to B, At equilibrium, 0.006-Q _0.01+O 4ne,(0.02) ~ 46 (0.04) = 0.000 67 wc ras Positive charges flow from sphere of higher surface potential to sphere of lower surface potential until the two. spheres have the same surface potential. = £0:006=0.000 67210" | 5 soy 4 x8.85x10 ? x 0.02 {© 2010 Times Publishing (Hong Kong) Limited -12- Physies Suveess Key for HKDSEE 4 Og, UT electrostatics o gm 5 A positively charged metal sphere A of radius 2 cm is joined by a conducting wire to a distant uncharged ‘metal sphere B of radius 1 cm. Find the ratio of the surface charge density on sphere A to that on sphere B. At equiltbriune, The student finds the ratio of the charges instead of the surface charge densities on the ‘two spheres. ‘At equilibrium, Qy 4me,(0.02) ~ 4n€,(0.01) Q 2 ‘The surface charge density on sphere A is rn 2 If charges Q ere uniformly distributed on the surface of a conductor, then the surface Q, charge density o is defined as -— 4a(0.02)" and that on sphere B is O% 4a(0,01" Hence, the ratio is where A is the surface area of the conductor . i {Qh 2 4a{0.02)“ 4n(0.01Y 1 2 © 2010 Times Publishing (Hong Kong) Limited -13- Physies Success Key for HKDSEE 4 24st ntrsontin go (25 items, 1 mark each) 1 am ‘A small metal sphere is suspended by an insulated thread as shown in the above figure. When a positively charged rod is brought near the sphere, the sphere is attracted by the rod as shown in the figure. Which of the following may happen if @ negatively charged rod is brought near the sphere? (1). The metal sphere is repelied by the rod. 2). The metal sphere is atracted by the rod. (3). The metal sphere remains stationary at the lowest position, AL (I)only B. (3) only €. (1) and @) only D._(1)and @) only ‘Two insulated uncharged metal spheres X and J Yare in contact with each other. A positively charged rod is brought near X without any contact while sphere is earthed as shown, aes At steady state, which of the following descriptions is / are correct? (1) Sphere X is negatively charged (2). Sphere Yis positively charged. (3). Electrons flow from sphere Y to the earth, {© 2010 Times Publishing (Hong Kong) Limited “14- Page total AL (only B. ()only C. (1)and Q) only D. Q)and (3) only oO Two insulated conductors X and ¥ are hung side by side in contact. A positively changed sphere is brought near Xas shown in the following figure. If Y is momentarily earthed by a finger, and then the positively charged sphere is removed, wl happen to Xand ¥? (1). Electrons flow from X'to ¥. (2) They both carry negative charges. (3) They separate from each other. A. (only B, (1)and 3) only C. @)and @) only D. (1), 2)and (3) The magnitude of the electrostatic force between ‘wo point charges is F. If the separation between the two point charges is doubled, find the new magnitude of the electrostatic force. A F a a F or P 7 oO Physies Success Key for HKDSEE 4 Page total 5 y “@ 34. 2a a Four patticles earrying charges +9, -39, +g and —2g ate placed at the vertices of a square of side Find the magnitude of the net electric force acting on the ~3g particle, AL zero 3@/2 +g? 4ne,° 6 Two equally charged particles, A and B, are J) separated by a distance of 4 em as shown in the figure, If third charged particle C is placed at the midpoint between the two spheres, the net electric force acting on C is zero. Which of the following statements is/are correct? o 2 6, <4 em ——> (1) and B must have the same sign of charges. (2) If Cis displaced from the midpoint towards 4 and then released from rest, it will move towards 4. @) If Cis displaced upwards from the midpoint and then released from rest, it will move towards the midpoint. (i) only Q) only (1) and (3) only (@)and (3) only ‘© 2010 Times Publishing (Hong Kong) Limited 1. yw y “1B Cet] Eker $y }$—_$§$ 2-1 ‘Two equally changed metal spheres of changes +g are fixed at two different positions. The separation between the centres, 4 and B, of the two spheres is 2r as shown in the figure. X is a point on the perpendicular bisector to the line joining 4 and B. Find the electric field at X when 0 = 60°. a Se Tore, B. c. The figure shows two changed particles of charges +Q and -@ respectively. X is a point on the perpendicular bisector to the line joining the two particles. Ifa third charged particle of charge +0 is placed around X so that the net electric field at X becomes zero, which position should it be placed? Al Ba 2 D4 Physics Suoeess Key for HKDSEE 4 oo Electricity and Maguelism ag Page total 9. c Dd. yd & e Four charged particles are placed atthe vertices of a thombus of side r as shown in the figure. One of the interior angles of the shombus is 60°. Find the net electric field at the centre O of the rhombus | 11. 9 due to the four charged particles. y a AL zero : a 2m ik a @ avid. c fae Three point charges are placed at the three a comers of an equilateral triangle of side r as >» 4 shown in the figure, All the point charges have the Srey? im same amount of charges q. Find the total electric potential energy stored in the triangle. 10, ‘The following figure shows how the electric J) potential ¥ in a region varies with the A, 3 & displacement r from a fixed point 0. ae y 3¢ Bey a = my . pv, dae Which of the following graphs best represents the * aney a) variation of the electric field strength E with r’? B. e 1© 2010-Times Publishing (Hong Kong) Limited -16- Physics Sueeess Key for HKDSEE 4 Page total 12, py Go 3 Ga- 5 ‘© 2010'Times Publishing (Hong Kong) Limited ‘The above figure shows a positively charged metal sphere 4 with radius R. Which of the following statements is/are correct? a 2 8) The electric potential due to the sphere is inversely proportional to r where r is the distance from a point to the centre of the sphere with r> R. The electric potential is zero inside the metal sphere When another positively charged metal sphere B is moved towards 4, work has to be done on B. (only Q)only (1) and @) only Q)and @) only LJ The electric potential at a distance r from a point charge is V. What is the electric potential at a distance 2r from the point charge? A B. AIS NIN g[s 4. Op UML Ebectrosiatics o ‘Two electrons initially separated by a distance of 5.0 x 10 m are released from rest. Find their instantaneous speeds when their separation increases to 8.0 x 10* m. Given that the permittivity of vacuum is 8.85 < 10 Fm! the mass and the charge of an electron are 9.1 x 10" kg and 1.6 x 10°" C respectively. A. B. c D. 15. y 436m s* 616ms* 918ms! 1380ms" ‘The above figure shows a pattern of electric field lines in which points 4 and B have the same electric potential. Which of the following statements is/are correct? wo 2 @) BP D. “17- When a charge is projected away from point B, it will move towards 4 with a constant velocity. When a charge is released from rest at point A, it wll be accelerated towards C. The electric field strength at point C is higher than that at point B. (i) only G) only (1) and (2) only (2) and @) only O Physies Suceess Key for HKDSEE 4 24 cect ent vorain 0 16. W B cB In the above figure, the solid lines represent part of an electric field due to a fixed point charge Q {not shown in the figure), A charged particle g, subjected only to electrostatic force in the field, travels along the dotted curve as shown. Which of the following conclusions ean be drawn’? ‘A. q is travelling from X'to ¥. B, The charge of q has the same sign as that of Q. C. qhas a greater speed at X than that at Y. D. The electric potential at X due to is higher than that at ¥. 17. yb In the above figure, the solid lines represent part of an electric field due to a fixed point change (not shown in the figure). A charged particle g. subjected only to electrostatic force in the field, travels along the dotted curve shown. Which of the following conclusions can be drawn? (1) qdeceterates first and then accelerates. (2). The work done on g by the electric force is positive. () The sum of kinetic energy and electric potential energy of g is conserved A. (I)and (2) only B. (1)and (3) only C. (2)and (3) only D. (1), @)and (3) {© 2010 Times Publishing (Hong Kong) Limited 18. yy 20. y -18- Page total oR A positively charged particle travels along a curve from P to Q in an electric field as shown in the above figure. The particle is subject to electrostatic forces only. Which of the following statements are correct? (1) If the particle is released from rest in the electric field, then the curve is also an electric field Tine. (2) Ifthe particle moves with constant velocity, then the curve lies on an equipotential surface, (3). IF the particle moves with acceleration, then the potential at P is higher than that at Q. A. (1) and (2) only B, (1) and(3) only C. (2) and (3) only D. (1), @)and (3) ‘Two horizontal metal plates are connected to @ 6V d.c. supply, The separation between the two metal plates is 1 em, Estimate the electric field strength between the metal plates. A. 300m" B. 450Vm" Cc. 600V mr" D. 1200Vm A negatively charged particle is placed in the middle between two horizontal metal plates connected to a d.c. supply as shown in the figure. It is found that the particle remains stationary when it is released from rest. If the separation between the two metal plates is doubled and the charged particle is also placed in the middle between the metal plates, what will happen to the charged particle when itis released from rest ‘again? Physics Suocess Key for HKDSEE 4 Page total ‘A. ‘The charged particle remains stationary B. The charged particle accelerates upwards. C. The charged particle accelerates downwards. 1D. The charged particle performs a penjesie mation. CL) u Og CHL sess iy 2) Zz A ‘Two distantly separated metal spheres, A and B, are initially charged to +0 and 420 respectively. The radii of A and B are R and 48 respectively. The two metal spheres are then joined by using a Jong conducting wire as shown in the figure. Find the charges on sphere A at steady state a. 4 * is) eer = ‘Two parallel metal plates are connected to a dc. supply as shown in the igure. Xi a point inside the metal plates with a distance r from the lower plate. Which of the following statements are correct? (1). The electric field between the metal plates is almost uniform. @) The electric potemial at ¥increass with @)_ Ifthe lower metal plate is earthed, it becomes neutral. ‘AL (I)and @) only B. (1)and(3) only C. @)and@) only D. (1),@)and) 2. y a fi) A positively charged metal sphere 4 of radius joined by a long conducting wire to an uncharged ‘metal sphere B of radius 2r. Find the ratio of the charges on sphere A to that on sphere B at steady O {© 2010 Times Publishing (Hong Kong) Limited -19- ‘Two positively charged metal spheres, 4 and B, are joined by a long conducting wire as shown in the figure. The radii of 4 and B are R and 2R respectively. Find the ratio of the surface charge density on sphere d to that on sphere B. A i Bw tot ce fa Do Physics Success Key for HIKDSEE 4 O94 cerned vari 0 Page total 25, (1) The ratio is independent of the tension in the my string 2). The ratio is proportional to tan 0. (3) The ratio is proportional to £. A. (only B. (3)only C. (1)and (2) only D. (2)and (3) only A positively charged ball is suspended by an insulated inextensible string. When a horizontal uniform electric field £ is applied to the ball, the bal is displaced by an angle @ to the vertical as shown in the figure. Which of the following statements about the charge to mass ratio of the ball is/are correct? (G items) Step 1: A positively charged rod is brought near an insulated neutral metal sphere. J Step 2: The metal sphere is momentarily earthed, Step 3: The charged rod is removed away from the metal sphere. Step 4: The metal sphere is momentarily earthed again, (@) What are the net charges induced in the metal sphere in the above four steps respectively? (3 marks) (b)_ If the metal sphere is earthed by using conducting wires, describe the lowing of charges during the earthing process. (2 marks) (Total: 5 marks) {© 2010 Times Publishing (Hong Kong) Limited Physios Suocess Key for HKDSEE 4 oe, 1 East le Page total 2 y ‘Two positively charged metal spheres, A and B, are suspended from two insulated threads of equal length of 1 mas shown in the figure, The charges on A and B are +10 xC and +25 uC respectively. In equilibrium, the thread connecting to sphere 4 makes an angle 30° with the vertical and the thread connecting to sphere 8 makes an angle 60° with the vertical. Find the masses of spheres 4 and B respectively. Given that the permittivity of vacuum is 8.85 x 10°"? F m' and acceleration due to gravity g=10ms™. (Total: 5 marks) 3. yD 3 [5 A positively charged particle of charges +40 and a negatively charged particle Y of charges -Q are separated by a distance of | m as shown in the figure, (a) The net electric potential is zero at point A. Find the distance 7; from point A to particle Y. (2 marks) (b) The net electric field strength is zero at point B. find the distance r; from point B to particle y. (3 marks) (Total: 5 marks) 2010 Times Publishing (Hong Kong) Limited 21+ Physics Success Key for HKDSEE 4 24 cei nt ean 0 j — ‘ov 100m ox Page total ‘Two parallel metal plates have a potential difference of 10 V with the lower plate being earthed as shown in the figure. X isa point inside the metal plates with a distance r from the lower plate. (a) ) Neglecting edge effect, sketch on the following space the equipotential ines at 2 V intervals and the electric field lines between the two metal plates. (2 marks) ‘Sketch two graphs respectively to show how the potential and the electric field at X vary with r i the separation between the two metal plates is 10 em GB marks) (Total: 5 marks) (© 2010 Times Publishing Hong Kong) Limited -22- Physies Suceess Key for HKDSEE 4 Unit | eecios se Page total j t00V Two parallel metal plates are connected to a dc. supply of 100 V with a separation of S em as shown in the figure, When a negatively charged oil drop X of mass | g is placed between the metal plates, it will be accelerated upwards with an acceleration of 1 ms, Find the amount of charges stored in the oil drop. Hence, find the total K.E. gain of the oil drop when it moves from the midway between the metal plates to the upper plate, Given acceleration due to gravity g= 10 ms*. (Total: $ marks) yw ‘The above figure shows how the electric potential V varies with the displacement r from a fixed point P. A proton is placed at the position of r= 4 m and then released from rest. Describe the motion of the proton and find the corresponding accelerations or decelerations while it is moving from r= 4 m to r=0 m, (Given: mass of proton = 1.67 « 107” kg, charge of proton = 1.60 * 10" C) (Total: 5 marks) © 2010 Times Publishing (Hong Kong) Limited -23- Physics Suovess Key for HIKDSEE 4 RE crv i nin go Page total (Baar Questions ¢ (Sitems) ‘A metal sphere carries a positive charge of 0.5 jC with its centre fixed at a point Q as shown in the figure. The radius of the sphere is S cm and the electric potential at infinity is taken to be zero. (Given: permittivity of free space = 8.85 «10 CN"! m=) (@)_ Find the electric potential on the surface of the sphere. (2 marks) (b) The electric potential drops to half of the value obtained in (a) at point B and ry is the distance from point B to O. (i) Find the distance rp (marks) (ii) Find the electric field at point B. (marks) (©) @) Sketch in the following graph the variation of the potential V at all values of r where r is the distance measured from 0. (G marks) viv ° rlom (© 2010 Times Publishing (Hong Kong) Limited 24- Physics Suocess Key for HKDSEE 4 Og Ltrs Page total (ii) Sketch in the following graph the variation of the magnitude of the electric field strength E atall values of r where r is the distance measured from O. (GB marks) ENV? o rem (Total: 12 marks) 2. The electric potentials Vat different distances r from an unknown small charged sphere are tabulated as below. Lo 12 14 16 18 1.80 -1.50 129 “113 1.00 (a) Explain the physical meaning of the negative sign of the electric potential. (2 marks) (b) @ A0.10 kg mass of positive charge 0.25 C is projected radially away from the sphere with a velocity of 2.0 ms" atyr= 1.0m. Find its velocity when it moves tor= 1.8m, (3 marks) (ii) Ithe positive charge in (b)(i) needs to be projected to infinity, find its minimum escape (2 marks) ‘© 2010 Times Publishing (Hong Kong) Limited -25- Physies Suecess Key for HKDSEE 4 eo Electricity and Magnetism. Page total (©) By plotting a suitable straight line graph, estimate the net charge (Given: permittivity of free space ey = 8.85 « 10" CN"! ried by the sphere. (5 marks) (Total: 12 marks) ee Four particles of equivalent charges +Q are placed at the four corners of a square ABCD of side d as shown in the figure. O is the centre of the square and the dotted line is the central axis perpendicular to the square through O. A negatively charged particle is placed on the central axis and then released from rest. © 2010 Times Publishing (Hong Kong) Limited Physics Success Key for HKDSEE 4 Og HL stems iy Page total (a) If X carries a charge of -O, find the electrostatic force exerted on X when itis at a distance r from O. Express your answers in terms of Q, rand d. (6 marks) (b) has a mass of | x 10" ke and the length of the diagonal of the square is AC = 1.4.em. If =2,uC and the initial position of X is r=2.4 em, estimate the speed of X when it reaches O. ‘Take the electric potential to be zero at infinity. (Given: permittivity of free space e = 8.85 10" C°N" mr?) (4 marks) (Total: 10 marks) 4 Ww switch uniform lever Mu <—— t +e, tiny metal prot ‘The figure shows an experimental set-up to measure the electric field strength between two parallel metal plates. A tiny metal plate M is attached to one end of a uniform lever. Some negative charges ~g are deposited to M. Before closing the switch, the lever is balanced about the pivot when the collar of mass 20 g is at a position dy em from the pivot. After closing the switch, the position of the collar has to be adjusted in order to restore the balance of the lever. \© 2010 Times Publishing (Hong Kong) Limited hysies Success Key for HKDSEE 4 2 Part 4 electricity and Magnetism Page total (a) How should the position of the collar be adjusted in order to restore the horizontal position of the lever? Explain briefly. (2 marks) (b) Label all the forces acting on the lever in the above figure when the switch is closed and indicate the senses of all the moments about the pivot produced by these forces. (3 marks) (©) The new position of the collar is d em from the pivot when the switch is closed. The experimental results for different amount of charges -q are as follows: ae | 10 2.0 3.0 4.0 5.0 ‘lem 98 9.02 347 798 752 (i) Explain why the laboratory should be dehumidified before performing the experiment. (1. mark) Gi) Plota graph of g against d. G marks) {© 2010 Times Publishing (Hong Kong) Limited -28- Physics Suocess Key for HKDSEE 4 Og 2 sles Sy Page total (tii) From the graph plotted in (e)(i), find the electric field strength between the parallel ‘metal plates and the initial position dp of the collar. Given = 0.7 m. G marks) (Total: 12 marks) % DD 20 om —___ Fig. (a) The figure shows that two parallel metal plates, and ¥, of length 20 em each are connected! to a d.c. power supply. (a) Sketch the electric field pattern between X'and ¥. The direction of the field should be shown. (marks) (© 2010 Times Publishing (Hong Kong) Limited -29- Physics Success Key for HKDSEE 4 RA sty nt optim Page total (b) An electron (with mass m and charge -e) is projected into the electric field with an initial horizontal velocity vy along the central axis between the parallel metal plates and finally hits one of the metal plates. (@ Sketch the path of the electron in the electric field in Fig. a). (marks) ii) Ifthe vertical displacement of the electron is d when it hits the metal plate, show that the horizontal displacement s of the electron is given by = 2dy, | ev where V is the e.m-f, of the dc. supply. State any assumption(s) made in deriving the above relationship. (4 marks) (ii) IE V=6V, d= 1 cmand yy = 7 * 10m s", find s. Given that the charge to mass ratio ofan electron is 1.76 « 10" C kg" (i mark) (iv) Ifa proton is used instead of the electron with the same initial horizontal velocity v9, describe and explain the change(s) observed. GB marks) (Total: 11 marks) © 2010Times Publishing (Hong Kong) Limited Physi Suecess Key for HKDSEE 4 oe. U1 Skersates iy Page total aa: item) 4. (@)_ What isthe physical meaning of an electric field” Explain why electric field lines never cross. (3 marks) a (b) Describe, with the aid of a diagram, an experiment to show the electric field pattern between two parallel straight electrodes. (6 marks) © 2010 Times Publishing (Hong Kong) Limited -a1- Physi Success Key for HKDSEE 4 aes Electricly and Magnetism ase Page total (©) The following figure shows an isotated positively charged metal sphere. Draw in the figure (the electric field lines representing the electric field patter (3 marks) (i) three equipotential surfaces with equal electric potential difference between consecutive surfaces; around the sphere. (3 marks) (Total: 15 marks) ‘© 2010-Times Publishing (Hong Kong) Limited -32- Physics Success Key for HKDSEE 4 Physics Success Key for HKDSEE 4 Name: Class : ty Date = 3 is Circuits and Domestic 5 2 Electricity | 7 = ie) 8 2B 3 © Examination Analysis Past Examination Paper Analysis .. Examination Trend .. © Useful Formulae .. © Examination Scoring Techniques © Common Mistakes ... @ Revision Test Multiple-choice Questions .. Short Questions... Structured Questions Essays... Total AS ‘Total marks 8 PU Cea ac Cun a! Part 4 electricity and Magnetism | @yshamination anatysis — Past Examination Paper Analysis (a) Multiple-choice Questions 4 HKCEE HKAL ape Year Paper | Question No. | Year Paper | Question No. Electric circuit 2001 nl 29, 30 2004 i 28, 29, 30 2006 I 23 i we] w [os Resistance and Ohm's law 2003 i 32,33 2008 TA 15 2004 tl 31 2005 IB 39, 40, 41 2007 IB 41 Series and parallel circuits 2003 u 34 1999 nA 7 2005 nA 18, 19 2001 nA 22 2006 vA 26 2002 TA 24 2007 nA 18, 20 2004 nA 20,21 Simple circuits 2007 nA 21 1999 mA 16, 19, 23 2002 nA 25 2004 nA 22,23 2009 nA 25,28 © 2010 Times Publishing (Hong Kong) Linsited Physies Success Key for HKDSEE 4 Gg UM 2 circuits and Domestic Electricity HKCEE Topic - ~ = Year | Paper | Question No.| Year | Paper | Question No. Electric power 2003 | it 36 1999 | A 20 2004 | on 32,33 | 2003 [ ma 36 2005 | uA 20 2008 | nA 16 2006 [uA a 2006 | UB 35 2007 | mA 2,23 2008 | mA | 20,22,23 2009 | mA 20, 41 Domestic electricity 2001 | 31,32 2003 | 35 2005 | ma 21 2006 | MA 25 2006 | mB 39, 40 2009 | mA 18,23 (b) Short Questions / Structured Questions Year | Paper | Question No. | Marks Content 1999 CE t 4 Connection of mains socket and calculation of cost of Paper 1A electricity To : 15 _ | Speci eat capacity, household wiring ring circuit) Paper IB and power rating. | = o 15 _ | Smples cies of potentiometer Paper 1B AL Measure the em. and the intemal resistance of a 2001 8 b), Paper 1p | 86) 0). 10 | patery AL Simple circuit of 2001 Paper IB Ma) 3 cE Power ra 2002 7(@), (b) 6 ee Paper IB E s fic ce y, latent hez ower i lec a 15 _ | Specific heat capaci, latent heat and power rating Paper 1B ©2010 Times Publishing (Hong Kong) Limited =3- Physics Suceess Key for HKDSEE 4 REE eit nd rnc 0 Year | Paper | Question No.| Marks Content El i r -alculation of of el soos | _ CE 10 15 _| Eletie power and calulation of cost of electric. Paper IB soos | _ 5 gg __ | Drawa circuit diagram and study the life of diferent Paper 1A cells. ind its heatin on | = 7 10 _| Study the circuitry of a hairdryer and its heating effect. Paper IB. cus | 7 1p | Aresperiment to investigate a seated box in which a Paper IB light bulb is connected to a variable resistor. cos | 3g _| Measure the intemal resistance of cell by using a Paper IA potential divider. cE Describe and calculate the energy conversion of a light 2007 7b), () 7 Paper IA 7 bulb cl : : ‘Advantages of connecting light bulbs in parallel. 2008 | Paper tA Connection of switch and consumption of light bulbs, (c) Essays Year | Paper | Question No.| Marks Content Describe an experiment to demonstrate the change in es0| atl 4 the terminal p.d. of a supply when delivering a current Paper IIB and explain how the internal resistance of the supply could be estimated. AL Explain why the potential difference across the terminals 2003 | paper IB sad + | cea bauery is smaller than tts emf ma Distinguish beoween e.m.f. and p.d. Connection of 2005 3a) 6 voltmeter and ammeter to measure the resistance of a Paper IB resistor ‘The variation of the potential difference along the path AL of'a simple circuit with two uniform resistance wires 2007 a 9 Paper IIB Ke connected in series, Describe how hedt is generated in # resistance wire, Examination Trend (a) Distinguish among electromotive force, terminal voltage and potential difference. (b) Study Ohms law and determine the factors affecting the resistance of a wire. (©) Compare series and parallel circuits in terms of the current and p.d. © 2010 Times Publishing (Hong Kong) Limited sae Physies Sueeess Key for HKDSEE 4 on U2 irs nd Domes lai | iy (@) Find, current, voltage and resistance in simple circuits. (e) Investigate the change of the current and p.d. ofa simple circuit before and after the switch is closed. (Describe how volimeter and ammeter should be connected in a circuit and explain the effects of their internal resistances on measurements, (g) Describe an experiment to measure the e.m.f, and the internal resistance of a cell (h) Apply P = V1 PR = . to solve problems, e.g. compare the rated value and the operating value of an electrical appliance @ Describe household wiring and discuss safety measures of domestic electricity i @iserut Formulae ‘Useful term Formula 1. Resistance oe Fi (R= resistance, V = potential difference, I = current) 2 Resscrisy RA ea (p = resistivity, A = cross-sectional area, ¢ = length) 3. Resistors connected in ae soca os RaR +R + Rt (R= equivalent resistance, R, = resistance of the nth resistor) 4. Resistors connected in Ry pacallel 5. Electrical power Pca (P = electrical power) (© 2010 Times Publishing (Hong Kong) Limited Physics Swocess Key for HKDSEE 4 ER sein ent vain go examination Scoring Techniques ———————_ 1. Connecting Series and Parallel Circuits (@_ Circuits in series DL et j——___ y____»} (a) The current flowing through each component is the same (b) The potential differences across each component are different which are given by V, = IR, IR, V3 = IRs. (© The total potential difference across all the components is equal to the sum of the potential difference across each individual component, i.e Vevev, V5 (ii) Cireuits in parallel ao (a) The potential difference across each component is the same, i Vy = Va= V5 = Vas (b) The currents flowing through each component are different which are given by Yoo 1, = Yan R R (© Atthe junction of the branches, the current is conserved, ic. Inh the © 2010 Times Publishing (Hong Kong) Limited Physics Suocess Key for HKDSEE 4 nit 2 circuits and Domestic leit 2. Measuring the Resistance of an Unknown Resistor @ Gi Resistor with large resistance ‘The cireuit should be connected as shown in the following figure: 1|-—_ Let V and I be the readings of the voltmeter andl the ammeter respectively. Then the experimental value of Ris given by Vv Reg 7 Tis the actual current flowing through resistor R, but V reads the sum of the voltage across the ammeter and that across the resistor, ie. V = Vq + Va. Hence, the actual value of R should be given by Vy Recwal = 7 Ree However, the experimental value is accurate for large value of Ras Vx >> Vj and hence Rp = Rest Resistor with small resistance ‘The cireuit should be connected as shown in the following figure: Let V and I be the readings of the voltmeter and the ammeter respectively. Then the experimental value of Ris given by v Rog = 7 Vis the actual voltage actoss resistor R, but I reads the sum of the current flowing through the voltmeter and that through the resistor, i. I= Ip + fy Hence, the actual value of R should be given by v Recast = ae Rew However, the experimental value is accurate for small value of R as ly >> ly and hence Rosy * Rest {© 2010 Times Publishing (Hong Kong) Limited -7- Physies Success Key for HKDSEE 4 a4 Electricity and Magnetism ga 3. Comparing the Electromotive Force (e.m.f.) and the Terminal Voltage of a Cell An actual cell has an internal resistance which can be considered as a resistor connected in series with an ideal cell as shown in the following figure: ne Ifa cell has an internal resistance, then the voltage supplied by the cel, called the terminal voltage, will be less than the e.m.f of the cell. The terminal voltage Van is given by Vag=e—Er where e is the e.m.f, of the cell, ris the internal resistance of the cell and Tis the current flowing through the cell. 4. Measuring the e.m.f. and the Internal Resistance of a Cell By varying the resistance of the theosta, different currents can be obtained. Assuming that the resistance of the ammeter is neglible, we have e=KR+r) > Re a A straight line is obtained by plotting R against - The emf. and the internal resistance of the cell can then be found from the slope and the y-intercept of the straight line respectively. © 2010 Times Publishing (Hong Kong) Limited Physics Suceess Key for HKDSEE 4 axe 2 Cis an Dometic Bcc |p 5. Measuring the Efficiency of a Battery Consider the above circuit. The output power is Pou = PR= (gs) Rer (R+ry The power delivered by the battery is € ie Panels e| © e {2;] Rer Hence, the efficiency ofthe battery is Fw R Fes 100% = x 100% The variations of P,,, Pj. and 7 with Rare shown below. 4 The efficiency approaches 100% when the load is very large, but the power delivered by the battery becomes very stall. {© 2010 Times Publishing (Hong Kong) Limited -9- Physics Suocess Key for HKDSEE 4 on 4 electricity and Magnetism ae Xx © common mistakes GG» 1A20 a % Peace av In the above circuit, the current flowing through the 3 @ resistor is 1 A and the e.m.f. of the battery is 12 V. Find the potential difference across resistor Ry The pid. aovoss Fe, is the same as the p.d. aovoss the 3 @ resistor whish is Therefore, the p.d. Bovoss Fea is The student knows that the resistors have the oS. ee ‘same p.d. when they are connected in parallel, but he wrongly thinks that the p.d, across the parallel circuit is equal to the sum of the p.d ‘0ross each resistor in the parallel circuit. Correct solution The p.d. across Ry resistor is Ve, = 1X 3=3.V Therefore, the p.d. across Ry is The p.d. across the parallel circuit is the same. Vq=12-3=9V 9 —————______— ass the p.d. across each resistor in the parallel circuit ‘What will happen to the brightness of light bulb X when switch S in the above circuit is closed? Explain briefly the brightness of X will be deorensed as the current Rowing through X is decreased, The student has a wrong perception that the current lowing through X is decreased as the original current is divided into two paths when S is closed, {© 2010 Times Publishing (Hong Kong) Limited -10- Physics Suevess Key for HKDSEE 4 % Unit D circuits and Domestic Bletrit The brightness of X will remain unchanged as the p.d. across X remains unchanged. a The current; Rowing through X remains unchanged. However, the total current drawn from the battery is increased from /= Jy to = fy + iy when $ is closed 2A 6v,052 The above figure shows part of a circuit in which a current of 2 A flows in the circuit. The e.m.f, and the internal resistance of the cell is 6 V and 0.5 @ respectively. Find the pad. across the cell. rn The student simply uses the formula Ve
0.5 V. (3) When a resistor is connected in parallel with the electric component, the /-V charactetistic of the parallel circuit becomes a straight line passing through origin © 2010 Times Publishing (Hong Kong) Limited y -13- Op, UH 2 circuits and Domestic Electricity | o (only Qyonly (i) and (3) only 2) and (3) only pore ‘The diameter and the length of a uniform copper wire are 0.5 mm and 5 em respectively. If the resistivity of copper is 1.69 x 10 Q m, find the resistance of the wire, A, 430x 10° Q B. 1,08x10°Q C. 338x102 D. 169x102 uA s viv The above figure shows the variation of the current / flowing through a heating element with the voltage F applied across the heating element. How does the resistance of the heating element change with its temperature? A. The resistance of the heating element remains unchanged with temperature. B. The resistance of the heating clement increases with temperature C. The resistance of the heating element decreases with increasing temperature. D. Cannot be determined, DPhysies Success Key for HKDSEE 4 GRO cen ent vans go Page total 5 sa cs \ J 3 ) |F-{E it | = f Fig. (@) se é A a In the above cireuit, ammeter 4, reads 3 A when switch § is open, Find the readings of the two ammeters when S is closed. Assume that the ammeters are of negligible resistance, A Ay A ISA ISA BISA 3A Fig. (b) If the equivalent resistance of Fig. (a) is 3 Q, find the equivalent resistance of Fig. (b) © 3A 3A ie Py DB. 3A 6A im B. Q 3 8. et ac 12 y 2 a0 vp. 20 Oo a S |. Arrange the circuits below in ascending order of lea J equivalent resistances where R, > Rs > Ry: rev o R, {| "i In the above circuit, if ammeter 4, reads 1 A, a & what is the reading of 4? The ammeters are of | o+—__ }+__+—- negligible resistance and the internal resistance of @) : the battery is negligible. a 24 3 toh st. 2 @) . oe GIA D. 2A AL (),2),8) B. Q),(1).3) Cc 3).(D.2 D. Cannot be determined ‘© 2010 Times Publishing (Hong Kong) Limited -14- Pysies Suevess Key for HKDSEE 4 Page total 9 1 30 12. J o J 208 —o+ 109 t+—__}—$1 \ it In the above circuit, the current flowing through the 30 Q resistor is £. What is the total current ‘drawn from the cell? A 6 B. 5.51 ey D2 7 en» 2a . 62 62 y a {I In the above circuit, the voltage across AB is LV, Find the e.m.f, of the cell. Assume that the internal resistance ofthe cell is negligible. AL 3¥ B. 6V cov DB RV LJ n. I om 4 In the above circuit, all resistors are identical. If the voltage across BC is 6 V; find the voltage across AB. A 3 B. 43V c. 6V D. 75V a) {© 2010 Times Publishing (Hong Kong) Limited -15- 2 circuits and Domestic Electricity 1 & In the circuit above, what are the readings of voltmeters V; and V; if switch $ is closed? %, Vv, A OV ay B0V ov c 6v av D 6V ov ev {t- {I In the above circuit, the battery has constant emf, of 6 V and all the resistors have the same resistance of 2 @. If point B is earthed, find the electric potential at point 4 when the switch is closed. AL 6V B. 4V c. 2Vv D. IV 1A 3¥,059 The above figure shows part of a circuit in which a cell delivers a current of 1 A to the cireuit. The ‘emf, and the internal resistance of the cell is 3 V and 0.5 @ respectively. Find the terminal voltage of the cell. Physies Success Key for HKDSEE 4 4 ceri nt arn 0 AD 35V Bo o3V ©. 278 D. 25Vv 18. y Rae —¥ 6a In the above cireuit, cell has an em.f, of 6 V and its internal resistance is negligible. Cell ¥ has an emf. of 3 V and its internal resistance is 1 Q. Find the current flowing through the 6 © resistor. AL zero B. OSA GIA D. 1SA O 16. 4 y Ae A cell of finite internal resistance is connected in series with a variable resistor R and an ammeter with negligible resistance as shown in the figure. By varying R, a series of ammeter readings, /, are taken and a graph of R against 4 is plotted. Which of the following gives the e.m.f. of the cell? A. The slope of the graph B. The inverse of the slope of the graph C.The.x-intercept of the graph D. The -intercept of the graph (© 2010 Times Publishing (Hong Kong) Limited 17. y 18, Wy -16- Page total iit | Lo The above figure shows a circuit diagram used to determine the resistance of resistor R. The reading of the voltmeter is V and the reading of the ammeter is [. Which of the following statements is/are correct? (1) The experimental value of 2 is given by f és 2) Fis inaccurate (3) The experimental value of 2 is higher than the actual value of R, A. (only B. @)only €.(1)and (2) only D. (2)and (3) only ih R ‘The above circuit is used to find the resistance of ‘a resistor R. Which of the following statements i are correct? (1). The ammeter reading records the actual current passing through R. (2) The voltmeter reading is smaller than the actual potential difference across R. (3) The experimental value of R found is smaller than is actual value A. (only, B. @)only ©. (1)and (2) only D. (2)and (3) only Physies Success Key for HKDSEE 4 Page total 19. 1 20, y : . wy In the above circuit, a voltmeter is connected across a variable resistor. Which of the following, graphs best represents the variation of the voltmeter reading with the resistance of the variable resistor? AD viv 1 “__»r/Qq 2. Bo oviv { y La {© 2010 Times Publishing (Hi -17- py eH 2 cies end Coma Ee 5a 52 oA 50 59 2 In the above eircuit, the battery has constant emf. of 6 V and has zero internal resis 4 high-resistance voltmeter is connected across 4 and B, it reads 3 V. Ifa low-resistance ammeter is connected across 4 and B, what is the ammeter's reading? AL 0A B. 02A C044 D. 064 im ance. If of a resistor R as shown above. Which of the following describe(s) the mistake(s) in the circuit? (1) The polarity of the ammeter is reversed. (2) The switch should be connected to the positive polarity of the battery, (3) The rheostat should not be connected in the circuit, A. (only B. Q)only (and Gy only D. (2)and@) only Physics Success Key for HKDSBE 4 O24 crtciv nd arn 0 n. ol @) )— In the circuit above, what are the changes of the readings of the three ammeters if switch S is closed? A A Ay ‘A. remains remains. remains unchanged unchanged unchanged B. remains inereases—_—increases unchanged C. decreases remains. remains unchanged unchanged D. becomes increases-—_increases zero 23, Two identical resistors R, two ammeters 4, and 4s J) of negligible resistance, a switch and a battery are ‘connected as shown in the following figure p>——| A U4). ‘What wil nappen tothe eading ofthe armetes if the switch is closed? A A; A incseeree a B. decmee C. decrees increases remains unchanged D. remains unchanged increases a {© 2010 Times Publishing (Hong Kong) Limited 24, y 25, yw -18- Page total {1| fa, A a s In the circuit above, all resistors are identical and the ammeters are of negligible resistance. What will happen to the readings of the ammeters if switch Sis closed? A Ay As ‘A. increases increases increases B. increases becomes zero increases C. decreases becomes zero increases D. increases increases becomes zero In the circuit above, all resistors are identical and the ammeters are of negligible resistance, If swith $ is closed, which of the following statements is/are correct? (1). The voltage across 4B becomes zero. (2) The readings of all the ammeters are increased. (3) The ratio of the readings of ammeters 4, to Ay is increased. A. (only B. (3)only C. (1)and Q) only D. 2) and (3) only ey LJ Physics Suevess Key for HKDSEE 4 Page total 26. y Three identical light bulbs are connected to a battery with negligible internal resistance as shown in the figure. If bulb Z burns out, which of the following is/are correct? (1) Bulb X becomes brighter, (2) Bulb Y becomes brighter. (3). The current drawn from the battery becomes smaller, A. (I) only B. (3) only C. (and (2) only D. (2)and(3) only H 21, y Three identical light bulbs are connected to a battery as shown in the figure. If bulb Z burns out, \witich of the following statements is/are correct? (1). The reading of 4) decreases and the reading of 4; remains unchanged (2) The brightness of X and ¥ increases. (3) X and Yhave the same brightness A. (only B. G)only (and Q) only D. (2)and (3) only {© 2010 Times Publishing (Hong Kong) Limited 28, y 29, y -19- Unit 2 circuits and Domestic Electricity 1 o In the circuit above, X, Yand Z are identical light bulbs. Which of the following statements is/are correct when switch S is closed? (1). The brightness of ¥ decreases, (2) Xand ¥ will have the same brightness. (3). The brightness of Z goes out. AL (only B. G)only C. (1)and @) only D. (2)and (3) only ga 12 Osa 290 The above figure shows part of a circuit which consists of some resistors. The magnitudes and directions of some of the currents are marked as shown, Find the current / flowing through R where R=2.0. AL 0.05.4. B. OLA C. 02a D. 034 Physics Suocess Key for HKDSEE 4 or Electricity and Magnetism age Page total 30. In the above circuit, the battery has constant em. yy of 3 V and has negligible internal resistance. The 22 2a, 28 resistors are all of resistance 2 Q. Find the current F AL OSA ; BIA c. 1SA D. 4S5A GSH Auestions 9 (Sitems) 1 y {6 x 10” electrons flow through the cross-section of a resistor in one minute as shown in the figure. (@)_ [each electron carries 1.6 x 10°" C of changes, find the current flowing in the resistor. (2 marks) (b) The voltage applied across the resistor is 80 V. (D _ Find the resistance of the resistor and the total energy dissipated in the resistor in one minute (2 marks) i) If the resistivity and the length of the resistor are 1.30 x 10° m and 1 em respectively find the cross-sectional area 4 of the resistor. (2 marks) (Total: 6 marks) {© 2010 Times Publishing (Hong Kong) Limited -20 Physies Success Key for HKDSEE 4 Page total 2. In.an experiment, the potential difference across a conductor is varied and the corresponding current flowing J through the conductor is measured. The results are tabulated as shown in the following table: Potential differenee HIV | 20 | 40 | 60 | so | 100 | 120 Current 1A, 1s | 26 | 34 | 41 | 46 | so (a) Plot against /. Hence, estimate the resistance of the conductor when = 50V. G marks) (b) Describe how the resistance of the conductor varies with current. Hence, deduce how the resistance of the conductor varies with temperature, (2 marks) (Total: $ marks) (© 2010 Times Publishing (Hong Kong) Limited -21- Physics Suecess Key for HKDSEE 4 E24 sev nd arn 0 Page total 3. Find the equivalent resistances ofthe following circuits: J @ (2 marks) 49 62 aa 20 o (marks) (Total; 5 marks) 4. J 3a 10 20 a \t In the circuit shown above, the current flowing through the 3 Q resistor is 2A. Find (2) theems. ofthe battery, (2maris) (© 2010 Times Publishing (Hong Kong) Limited -22- Physies Success Key for HKDSEE 4 Page total (b) the current flowing through the 1 Q und 2 Q resistors, (2 marks) (©) the voltage drop across the 2 Q resistor. (mark) (Total: $ marks) 5. The following figure shows the circuit used to find the resistance of an unknown resistor. y call_cal svt ‘vatiable resistor (a) Draw, inthe space below, the circuit diagram for the above circuit (2 marks) (b) The reading of the ammeter and the reading of the voltmeter are 1.21 A and 6 V respectively. Find the experimental value of the resistance of the unknown resistor. (1 mark) (©) Would the experimental value obtained in (b) be greater or smaller than the actual value? Explain briefly why. (2 marks) (Total: 5 marks) © 2010 Times Publishing (Hong Kong) Limited -23- Physies Success Key for HKDSEE 4 on A electricity and Magnets ge Page total GBUMLTETAvostions ¢ (items) 1 I aa © In the above circuit, point C is earthed. (a) Find the voltage across 4B before and after switch Sis closed (5 marks) (b)_ Sketch a graph to show the variation of electric potential with position along ABCDE when switch Sis open. Gmarks) wy (© 2010 Times Publishing (Hong Kong) Limited -24- Physies Success Key for HKDSEE 4 Umit D civeuis and Domestic Elciricty | Ped Page total (©) Sketch a graph to show the variation of electric potential with position along ABCDE when switch Sis closed. (3 marks) vv (Total: 11 marks) slide contact y rotary variable resistor Fig. (@) Fig, (a) shows a rotary variable resistor, A slide contact can be rotated on the rotary variable resistor with centre O. The resistance of the resistor is directly proportional to the angle of rotation 6. When the slide contact rotates to end B of the resistor, ie, @ = 270°, the resistance of the resistor has a maximum value of 13.5 Q. Find the resistance when @ = 60° and hence find the current flowing through the resistor (4 marks) £© 2010 Times Publishing (Hong Kong) Limited -25- Physics Suecess Key for HKDSEE 4 R24 serv nt tac 0 Page total (b) A student uses the rotary variable resistor in (a) to design a burglar alarm system as shown in Fig. (b). rotary variable resistor ‘A metal strip is stuck to the rim of a door and the metal strip contacts with end 8 of the rotary resistor when the door is closed. The metal strip will be rotated on the resistor when the door is open in the anticlockwise direction with centre O. The alarm system will be triggered to go off if the current flowing through the system is greater than a certain value. (® Explain how the alarm system works. (2 marks) i) The alarm system will be triggered to go off when the current flowing through the alarm system is greater than 1.0 A. Find the maximum angle that the door can be rotated without triggering the alarm system to go off. The alarm system itself has an internal resistance of 11 Q. marks) (ii) The student finds that the maximum angle that the door can be rotated is inereased after the system has been working for some time. Explain briefly why. (2 marks) (Total: 11 marks) © 2010 Times Publishing (Hong Kong) Limited 26 Physics Success Key for HKDSEE 4 Unit 2 circuits and Domestic Eetricy | o Page total 3. The resistance R ofa wire is given by y £ R= pL PA \where p is the resistivity of the material ofthe wire, ¢ and 4 are the length and the cross-sectional area of the wite respectively. An experiment is carried out to investigate the above relationship and the results are tabulated as follow: hm 2 4 6 8 10 6 6 6 6 Almm? | 0.05 | 005 | 0.05 | 005 | 0.05 On 02 03 04 RQ | 202 | 395 | 594 | 812 | 993 | 304 | 147 | 98 | 76 (@) Design a circuit diagram to measure the resistance of the wire and sketch the diagram in the following space provided. (2 marks) (b) Choose a suitable set of data from the above table to plot a suitable straight line graph to show that R is proportional to £. Hence, find the resistivity of the material of the wire used in the experiment, (S marks) (© 2010-Times Publishing (Hong Kong) Limited -27- Physies Success Key for HKDSEE $ Part 4 Elecvicity and Magnetism poe REA city and aston og Page total (©) Choose a suitable set of data from the above table to plot a suitable straight line graph to show that 8 is inversely proportional to 4. Hence, find the resistivity of the material of the wire used in the experiment, (S marks) (Total: 12 marks) Wy Resistance Temperature The above graph shows how the resistance of a resistor varies with temperature, Use molecular motion to explain why the resistance of the conductor increases with temperature. (2 marks) (b)_Presetting the temperatures of the resistor, the following table shows how the potential difference V across the resistor varies with the current J flowing through the resistor. Temperature/*C 2 | 40 | o || a | 10 Potential difference V/V | 20 | 40 | 60 | 80 | 100 (Current A Ottis | ia fete la Resistance /O ‘© 2010 Times Publishing Hong Kong) Limited -28 Physies Success Key for HKDSEE 4 Unit 2 circuits and Domestic leant Page total (® Plot R against T where Ris the resistance of the resistor, (GB marks) (ii) What can be concluded from the graph plotted in (b)(i) for the temperature range used in the experiment? Hence, find the relationship between R and 7: (GB marks) i) Estimate the resistance of the conductor at 0°C. Hence, find the fractional inerease in resistance for a unit rise in temperature with respect to the resistance at 0°C. (3 marks) (Total: 11 marks) © 2010-Times Publishing (Hong Kong) Limited Physics Success Key for HKDSEE 4 GRE stavros 0 Page total & Ww ¥ , Re B In the above circuit, a resistor is connected in series with a thermistor to a variable d.c. supply with negligible internal resistance. The following figure shows how the resistance Ry of the resistor and the resistance R of the thermistor vary with temperature, 2 40 «6 80 100 120 Temperature /°C (a) When the de. supply is set to 12 ¥, the potential difference across 4B is found to be 6 V, (@ Find the temperature of the resistor and that of the thermistor respectively. (2 marks) (i) Hence find the resistance of the resistor and that of the thermistor respectively. (2 marks) (b) When the dc. supply is set to 24 V, the potential difference across 4B is also found to be 6 V Dut the resistor is hotter than the thermistor by 20°C. (H Estimate the temperature of the resistor and that of the thermistor respectively (2 marks) {© 2010 Times Publishing (Hong Kong) Limited -30- Physies Success Key for HKDSEE 4 Ont 2 cits ond mesic Ee Page total (i) Hence find the resistance of the resistor and that of the thermistor respectively. (2 marks) (©) [fan identical resistor is connected in parallel with Ry as shown in the following figure and the d.c. supply is also 24 V, the potential difference across 4B becomes 12 V and all the components have the same temperature, Estimate the temperature and the resistance of the thermistor in this ease respectively. (4 marks) (Total: 12 marks) a (Litem) 1, The following figure shows an experimental setup to measure the internal resistance ofa cell. Dw © 2010 Times Publishing (Hong Kong) Limited -31- Physics Success Key for HKDSEE 4 & Part A Elecriciy and Magnetism aoe Page total (a) Sketch the circuit diagram of the above experiment, Indicate the polarities of the voltmeter in the circuit diagram. (5 marks) (b) What does the voltineter read in the experiment? (mark) (©) Use the following symbols E:the eam. of the cell, r: the internal resistance of the cell, R the resistance of the rheostat, V: the voltmeter reading, to describe how the e.m.f. and the internal resistance of the cell can be measured by the experiment. State any assumption(s) made. (© marks) (Total: 15 marks) (© 2010 Times Publishing (Hong Kong) Limited Physios Success Key for HKDSEE 4 Physics Success Key for HKDSEE 4 Name : Class : cy Date : Circuits and Domestic Electricity I Total as © Examination Analysis Same as Unit 2 , © Useful Formulae Z Same as Unit 2 2 © Examination Scoring Techniques Z _ Same as Unit 2 : Total 730 © Common Mistakes La Same as Unit 2 tl nl @ Revision Test _ 3 Multiple-choice Questions . : z Short Questions 7 : Structured Questions 11 Total 7 Essays... Meena ica Mead Tacs ae eure er Eletricty and Magntio gay @ Revision Testy (25 items, 1 mark each) ‘Which of the following statements is incorrect? “Ifourrent is 1 A, then A. the voltage drop across a resistor of 5 Q is sw B. the power dissipation in a resistor of 5 Q is sw C. the net amount of charges flowing through a circuit is 1 coulomb per second,” D. the energy supplied by a 3 V battery in Oo I second is 9° ‘The lithium cell in a mobile phone is charged for 8 hours at an average current of 350 mA. If the cell is charged by a dc. supply of 5 V, estimate the energy stored in the cell during this charging process. AL 12x 10°F B. 50x10) ©. 14x tot D. 20x10 Ea ov t-{-+——_| 100 109 sa In the circuit, what is the power dissipated in the 5.Q resistor? AL OW B. 72W Cc. 45W D. 4.05 W {© 2010 Times Publishing (Hong Kong) Limited y y Page total 1G In the above circuit, three identical lamps L), La and Zs of ratings “6 V, 9 W’ are connected to a 12.V battery. Find the current flowing through L. AC 1A B OLSA c. 2A D. 3.334, ‘Two light bulbs are marked ‘110 V, 50 W" and “110 ¥, 100 W’ respectively. If they are connected in series to a 220 V mains supply, what is the total power dissipated in the two light bulbs? A. 150W B. 133 Cc. 100W D. 75W by G In the above circuit, light bulbs L,, L; and Ls consume power at the ratio of 2: 1: 1. If the resistance of L; is R, what i the resistance of L;? eae 2 BR co iD. aR Physies Success Key for HKDSEE 4 Page total 1 y Three identical light bulbs are connected as shown in the circuit. Find the ratio of the power dissipated in L, to that dissipated in Ly %% meas teats an Conese arc ly (2) The current through Ly is smaller than the ccurrent through £). (3) The power dissipated in £, doubles the total power dissipated in Z, and L, ‘A. (1) and (2) only B. (1)and (3) only C. (2)and (3) only D. (1), (2)and 3) A 16:1 0, 4 — B. 4:1 y © 2:1 taal Dd. 1:1 ‘A motor is connected in series with a resistor and 8, Two electrical heaters X and ¥ are of ratings 12 power snppiy-axstown fu the abit. Ide J) “110 ¥, 40 W? and ‘110 V, 80 W" respectively. power delivers to the motor is 6 W when R= 6 Q. Which of the following statements about the two Find the eurreat through the resistor. heaters is/are correct? Gera (1) The rated resistance of Wis twice that of ¥ BRIA 2) X-will consume a power of 80 W when itis oa connected to 2 220 V mains supply. Oak G)_ Both heaters work at thei rated values when they are connected in series to a220V mains | 1. aa supply. y IFO5 A. (I)only B. G)only a . (1)and @) only D. @)and (3) only Fig. (@) 7 i vv j iG — Ls Ls = Fig. () ‘Three identical lamps Ly, Ly and L; of ratings “6 V, 12 W’ are connected to a 6 V battery as shown in the cireuit diagram. Which of the following statements are correct? (1) £; is working at the rated values whereas La and L, are not {© 2010-Times Publishing (Hong Kong) Limited In Fig. (a), the ammeter reads 1 A. In Fig. (b), the eam of the battery is 12 V and the ammeter reads 4A. Find the power dissipated in & in Fig. (a). A 3W B. 6W Cc OW > RW oO Physics Suecess Key for HKDSEE 4 qr Electricity and Magnetism a @ Page total 12, 4. Re y yD B x jo oa ® In the above circuit, the e.m.f. and the internal If the resistor Ry in the eireuit suddenly breaks, resistance of the battery is 6 V and 2 Q ‘which of the following statements is/are correct? respectively. If the elficiency of the power (0) Thesensiea paeting thst A i ceased dissipated in the variable resistor R is 80%, what : . is the resistance of R? @) The potential difference across Rs is increased. a 460 (3). The power dissipated in R, is decreased. = a A. (I)only 4 mM BR @)ealy D. 402 fe C. (I) and @) only 15. A d.c. supply of constant emf, and internal D. Q)and @) only J) resistance is connected to a variable resistor of 1 resistance R. Which of the following graphs best ee shows how the power P delivered to the variable y resistor varies with R? A B. P e If switeh $ in the circuit is closed, which of the \Z. L following statements is/are correct? i: (1) ‘The power dissipated in Ris increased, : : 2) The ratio of the power dissipated in R, to that in R, remains unchanged. R A 8 ated in R; becomes zero. oe> (1) and (3) only D. (2) and (3) only ‘© 2010 Times Publishing (Hong Kong) Limited -4- Physies Suecess Key for HKDSEE 4 Page total 16, Two cylindrical conductors, X and ¥, are of the JJ) samme material and the same mass. The length of Xis four times that of ¥. Ifthe two conductors are connected in parallel, find the ratio of the power dissipated in to that in ¥. A 1:16 Bos G wat Do 4st i UA wy 2 13) 1 os a oy The graph shows the /-V characteristic of two light bulbs X and Y, If-X and ¥ are connected in series to 4 50 V dc. supply of negligible internal resistance, estimate the power dissipated in light bulb x. A. 20W B. 30W Cc. s0W D. Cannot be determined as the ratings of the two light bulbs are not given. 18. vv y 240] 180) 120) x y 0] oa ae a A {© 2010 Times Publishing (Hong Kong) Limited 19, y 20. a. og 3. crs en one The above figure shows the /“1 characteristic of ‘wo light bulbs X and Y, If X and ¥ are connected in parallel to @ 120 V mains supply, estimate the total current delivered by the supply. A OBA B. 064A Gc LA D. 12A oO tev 1 ee 8 Lai 2 ‘Two light bulbs L, and L; of ratings °6 V, 9 W’ and. “6 V, 12 W’ respectively are connected in parallel as shown in the circuit. If the two bulbs are going, to be worked at their rated values, what should the resistance of the variable resistor R be? A. 72 B. 5.149 c. 3439 a D. 1719 Which of the following values is equivalent to one kilowatt hour? AL 3600W B. 3600) C. 36x 10°W D. 3.6% 10°F In a cable, different colours are used for different wires. Which of the following cortectly matches the colours with the wires? Live Neutral Earth A. Yellow/green Brown Blue B. Brown Yellow/green Blue C. Brown Blue Yellow/green D. Blue Brown Yellow/green Physics Suoeess Key for HKDSEE 4 tA electricity and Magnetism 8 cestrcinenes oo 2. J 23, © 2010 Times Publishing (Hong Kong) wire 4 ‘The above figure shows the three wires connected. 10 4 plug, Which of the following answers about the three wires is correct? Wire 1 Wire2 Wire3 A. carth five neutral B. earth neutral live C. neutral earth ive D. veiral tie cath O If the live and neutral wires in the plug of an electrical appliance are mistakenly interchanged, which of the following statements is/are correct? (1). The cireuit of the appliance remains at a high potential even when the switch is off. 2). The fuse in the plug will not blow if the appliance is overloaded. (3). The appliance will not work when the switch ison, A. (only. B. Q)only €. (1)and (3) only D. (2)and (3) only 24. I Page total The following table shows the aver: consumptions of three electrical appliances household. Appliance Rating | Duration Heater [2201 500W| 4 hours/day Television set_| 220 v, 300W | 2 hours/day Washing machine] 220 V, 1000 W] 6 hourvweek Estimate the cost of electricity used in February. Note: 1 kWh of electricity costs $0.86. AL $89.9 B. $179.6 c. $3034 D. $536.64 ‘Three appliances are connected to the same socket, Fuses are installed in each of the plugs of the appliances and also installed in the socket. If the rated value of the fuse installed in the socket is 15 A. Which of the following statements isiare correct? (1). The appliances are connected in parallel. 2) If the current through one of the appliances is larger than 15 A, then all the fuses will blow. G3) The total current through the appli should not exceed 15 A. A. (only B. (2)only ©. (Iand (@) only D._@)and@Q)only O Physies Success Key for HKDSEE 4 Op tntd Crt and Domestic eric 1 Page total ions (6 items) L y 5 le ‘Two cells E; and E; are connected to a light bulb of resistance 8 Q as shown in the figure. E, bas an e.m.f. and internal resistance of 6 V and 2 © respectively. £ has an e.m.f. andl internal resistance of 12 V and 2 £2 respectively. What is the power dissipated in the light bulb? (Total: S marks) 2, There is a light bulb of rating °6 V, 10 W" and a 9°V de. supply. A resistor is connected in series with the J supply and the light bulb so that the light bulb can be worked at its rated value. (a) Find the resistance of the resistor. (3 marks) (b) Find the efficiency of the circuit supplying power to the bulb. (marks) (Total: 5 marks) {© 2010 Times Publishing (Hong Kong) Limited “7 Physics Suocess Key for HKDSEE 4 oes Electricity and Magnetism a9 @ Page total 3 I evsew 6v,24W ‘Three light bulbs of ratings °6 V, 5 W", ‘6 V, 36 W" and °6 V, 24 W" are connected to a 6 V de. supply as shown in the figure. (a). Find the operating resistance of the ‘6 V, 5 W" light bulb (2 marks) (b) Which light bulb is the brightest? (GB marks) (Total: 5 marks) 4. I toov 38 AA fixed resistor X of resistance 50 is connected in series with a variable resistor Y to a steady dc. supply of 100 V as shown in the figure. Let P and R be the power dissipated in the variable resistor and the corresponding resistance of the variable resistor respectively (a) Express P in terms of 2. (2 marks) ‘© 2010 Times Publishing (Hong Kong) Limited Physics Suecess Key for HKDSEE 4 %9 Unit 3 circits and Domestic Electricity “ Page total (b) The following figure shows how P varies with R. PIW ° A/D 80 If another fixed resistor of resistance of 50 Q is connected in parallel with the resistor X, sketch a new graph of P against R. G marks) Piw ol RIQ (Total: 5 marks) 5. J — No— Connect a switeh and a fuse to the wire of the electric appliance and complete the above circuit diagram, The rated value of the electric appliance is ‘220 V, 1 100 W’. Which of the fuses, rated a3.A,5A and 7A, should be used? Explain why. (Total: 5 marks) {© 2010 Times Publishing (Hong Kong) Limited -9- Physics Success Key for HKDSEE 4 ERA steven ndrvanton go Page total 6. é y 114 heater ‘The output power of an electric heater is measured by using the above circuit, The results are tabulated as follows: uv 120 140 160 180 200 vA 43 46 48 5.0 5.0 ‘where Vis the reading of the voltmeter and I's the reading of the ammeter. {a) Plot P against * where P is the output power of the heater. (G marks) (b) Theoretically, the power dissipated in the heater is given by P = e where R is the resistance of the heater. Explain briefly why the graph plotted in (a) was not a straight line, (2 marks) {© 2010 Times Publishing (Hong Kong) Limited (Total: 5 marks) Dhysies Success Key for HKDSEE 4 Page total Gitems) L y @ Unit 3 cirewis and Domestic Electricity o ‘A student uses the above circuit to measure the e.tn.f.€ and the internal resistance r ofa cell, The ammeter reading / for different values of R are tabulated as follows: RID 10 12 14 16 18 HA 0.72 0.62 0.55 0.49 0.44 (a) Express & in terms of e, fand r. State one assumption made. G marks) (b) (Plot suitable straight line graph by using the above data in order to estimate ¢ andr. (4 marks) ‘© 2010 Times Publishing (Hong Kong) Limited “ne Prhysies Success Key for HKDSEE 4 Part 4. Ewetrity and Magneto REE eter nt Magni Page total (ii) Hence, find e and r from the graph plotted. G marks) (©) Ithe ammeter has a resistance of 0.5 @, which of the results obtained in (b)(i) should be corrected? Explain why. (2 marks) (Total: 12 marks) 2, A student is provided with two filament lamps 4 and B. Each is rated ‘12 V, 60 W*, but lamp 4 has 4 carbon J) filasnent and lamp B a tungsten filament. (@) Calculate the resistance of each filament lamp under normal operating conditions. (2 marks) (b)_ Fig. (a) shows the relationship between the current / and the potential difference I for the two filament lamps. WA (® Describe how the resistances of lamp 4 and lamp 8 vary with current. (2 marks) {© 2010 Times Publishing (Hong Kong) Limited 12+ Physics Success Key for HKDSEE 4 Unit 3 circuits and Domestic Electricity It fe] Page total (ii) Fig. (b) shows the two filament lamps connected in series to battery of em.f. 12 V and negligible internal resistance. nv F--— lampA lamp 8 Fig. (0) Estimate the power consumed by lamp A and lamp B respectively by using the graphs in Fig. (@). (5 marks) (©) How should the circuit in (b) be adjusted so that the two lamps can be worked at rated value? Explain briefly. (2 marks) (Total: 11 marks) 3, Dw ‘The following components are used to measure the resistance of alight bulb: ‘A battery, a switch, a voltmeter, an ammeter, a variable resistor and the light bulb. (a) Fig. () shows an incomplete circuit for the experiment. Use suitable circuit symbols to complete the circuit diagram to show how the components should be connected in the experiment. (3 marks) eS Fig. (a) '© 2010 Times Publishing (Hong Kong) Limited -13- Physics Success Key for HKDSEE 4 o= 4 eleteicity and Magnetison pag (b) What is the condition for the readings of the meters to be accurate? Explain briefly. Page total (2 marks) (©) A student performs the experiment and takes several readings. Fig. (b) shows one set of the readings obtained by the student. ‘mmaier(O=TA) _-Volmeter (0-200) Fig. (b) (@ What are the readings obtained in the measurement? (2 matks) Gil) Hence, find the resistance of the light bulb, (2 marks) (@)_ The rating of the light bulb is *200 Y, 200 W* @_ Find the operating resistance of the light bulb if it works at its rated value, (mark) (i) Account for the difference between the values obtained in (¢)(ii) and (d)(). (2 marks) (Total: 12 marks) © 2010 Times Publishing (Hong Kong) -14- Physics Suocess Key for HKDSEE 4 Gg, CM cinuis and Domestic Eletricty t Pod Page total 4. The following figure shows a ring circuit installed in a household, yw NLE @ el @ (a) Complete the above figure to show how the sockets are connected to the ring eireuit, G marks) () State two advantages of ring eireuit (2 marks) (©) A heater of rating +220 V, 2 000 W’ is connected to one of the socket and is switched on for five hours. Find the electrical energy consumed by the heater in the units of kWh, (2 marks) (@)_A.15 A fuse is installed in the live wire of the ring circuit, Should an electrical hairdryer of rating +220 V, 2 200 W" be connected to the other socket of the ring circuit if the heater in (©) is stil on? Explain why. G marks) (©) Explain why it is dangerous to put a plug into the socket if the plug is wet. (2 marks) (Total: 12 marks) {© 2010’Times Publishing (Hong Kong) Limited -15- oe” 4 electricity and Magnetism poe Page total 5. (a) An electric kettle connected to a 220 V a.c. supply can heat up 1 kg of water from 20°C to 50°C in Ww 3 minutes. The rated value of the heater is “200 V, 1 000 W’. (Find the useful output power of the kettle, Given that the specific heat capacity of water °c". (2 marks) (i) Find the input power of the kettle, State one assumption made, Hence, find the efficiency of the kettle (3 marks) A student says that the efficiency would be higher than that obtained in (i) if the kettle is worked at its rated value. Do you agree with him? Explain your thought, (2 marks) ) x yoo ze 7 ‘The above figure shows the electric kettle as mentioned in (a). A 7 A fuse is installed in wine Yof the electric kettle. (Name wires X, Yand Z. G marks) (i) Lf there is current leakage at point P and the leaking current is 4 A, will the fuse blow? Explain briefly. er) (Total: 12 marks) ‘© 2010 Times Publishing (Hong Kong) Limited -16- Physies Success Key for HKDSEE 4 Unit 3 circuits and Domestic Electity t o Page total a; (item 1, Fig. (a) shows a household electrical wiring circuit. The mains cable (containing live and neutral wires) is J)) connected to a consumer unit via an element X. At the consumer unit, the wires branch out into a number of parallel main ‘consumer rit ‘switch + sit onl | ‘connected to = lighting cut x . = Connected to TH ing circuit ‘connected to only one lectc 1s eg ete epplance main fuse NL Fig. @) (@) Name the element X. What isthe function of X? (2 marks) (b) Why are the branches connected in parallel? (mark) (© 2010 Times Publishing (Hong Kong) Limited -17- Physies Success Key for HKDSEE 4 ee 4 Electricity and Magnetism joey Page total (©) One of the branches is connected to a ring circuit. Sketch a circuit diagram to show how the ring circuit is connected, State two advantages of ring circuit (4 marks) (@)_ One of the branches is connected to only one electrical appliance, Explain briefly why. (2 marks) (©) One of the branches is connected to a lighting circuit. How are the lightings in the lighting circuit connected? Give two reasons for this connection. (3 marks) {© 2010 Times Publishing (Hong Kong) Limited -18- Physies Suocess Key for HKDSEE 4 Unit 3 circuits and Domestic Electricity I Fe] Page total (0) State three ways to save domestic electrical energy. G marks) (Total: 15 marks) ‘© 2010 Times Publishing (Hong Kong) Limited -19- Physies Suecess Key for HKDSEE 4 Stn Cen Om mn ae) © Examination Analysis Past Examination Paper Analysis Examination Trend .. © Useful Formulae © Examination Scoring Techniques ..... © Common Mistakes @ Revision Test Multiple-choice Questions . Short Questions. Structured Questions Essays... Dee ca ey Name: Class: Total Total 2s ‘Total AS ‘Total marks Ais on 4 electricity and Magnetism Qe gkamination analysis Past Examination Paper Analysis (a) Multiple-choice Questions HKCEE HKAL Topic Year | Paper | Question No.| Year | Paper | Question No. Magnetic force and magnetic 1999 | 1A 27 Geld 2001 | mA a 2002 | mA 21 Magnetic effect of electric 2001 | 33 1999 | A 35 ‘current 2003 | 37 2000 | mA 25 O 2004 | 34 2002 | mA 29 2005 | MA 2 2003 [mA 30, 31 2008 | 1B 8 2003 | mA 29 2009 | mA 21 2006 | MA 18 2009 | mA 29 Electromagnet 2001 | 36 Force on current-carrying 2007 | UB 40 2005 | mA 32 conductor in magnetic feld | 2008 | 1A 19, 42 2009 | MA 30 2009 | mA 2 Simple dc. motor 2005 | MIA 2B 2000 [ua 28 2007 | UB 42 2002 | mA 30 a Hall effect 1999 | A 30 2000 | MIA 26,27 200+ | mA 26 Induced voltage and induced | 2004 [UL 35 1909 | MA 36 current 2005 | 1B 2 2001 | mA 28 2008 | IIB 39,40 2002 | nA 27,28 2009 | UB 39 2004 | MA 28 2006 | MA 19,21 Faraday’ law of 2001 | 35 2005 | WA 19 electromagnetic 2006 | UB 36 2009 | IA 32 induction oF | ie 3 (© 2010 Times Publishing (Hong Kong) Limited Physics Success Key for HKDSEE 4 git sternal HKCEE HKAL Topic - —— = Year | Paper | Question No. | Year | Paper | Question No. Generator 2004 | mB 36 2009 | UB 40 Eddy current 2008 | mA 18 Alternating currents 2000 | mA 35 2oor_| ua 30 2005 | MIA 37 Transformer 2003 | 38 2004 | TA 7 2006 | UB 37,38 | 2006 | A 5 2007 |B 4 2008 | MA 19,35 2008 |B a8 2009 |B 4 Transmission of electric energy | 2001 | 34 2005 | IB 8 (b) Short Questions / Structured Questions Year | Paper | Question No.| Marks Content 199 | _ : 5 _ | The working principle of microphone, Paper 1A 19 | _ & : 1y_| Moving charge in a magnetic fied Paper 1A cE Electric motor. 2000 | pacer A 6 6 ee 7 15. | The working of 2 cooker by using 2 transformer and Paper 1B explanation on the labelled values of the cooker. ae ; 1 | Magnetic feld produced by two long straight parallel Paper 1B wires, {© 2010 Times Publishing (Hong Kong) Limited -3- Physies Suecoss Key for HKDSEE 4 R74 exec nt naontion gg Year | Paper | Question No.| Marks Content Movement of charges in a metal bar moving in a zo | _ at : 15 _ | unlform magnetic field. Study an experiment to test an Paper IB idea for generating electricity in the upper ionosphere by electromagnetic induction. oe : 5 __| High voltage transmission of electrical energy. Paper 1A “ Study the 1+ graph of an electric motor. Calculate the 2001 13 | torque acting on the coil, the back e.m.f, developed Paper IA across the coi) and the efficiency of the motor. a003 | _ &E : ¢ | Convert the motor of an electric fan to a simple Paper LA generator. 208 AL - 11 _ | Electromagnetic induction in a coil placed in a Paper 1A changing magnetic field. | a 15 __ | The working of an electric toothbrush and the use of a Paper IB transformer to charge up the brush. Study the circuit breaker installed in a domestic circuit mor | 2 : 15 _ | Gleulate the corresponding magnetic flux density and Paper IB the induced e.m.f. The differences between fuses and circuit breaker. a005 | _ a 7 | Amexperiment to investigate a transformer Paper IB i ‘An experiment to investigate the relationship between 2006 ? 6 the strength of an electromagnet and the number of Paper IA turns of its coil. ee Z 13_ | Avexperiment to study the force induced on a current Paper 1B carrying wire in @ magnetic field soe | 7 g__| Generating an ac. by rotating a bar magnet between Paper IB two solenoids. or | : 11 | Measuring the magnetic field by using a simple current Paper IA balance, a8 cE : 5 Simple d.c. motor. Paper 18 a i: | Hduced curent ina solenoid while a magnet is falling Paper IB through the solenoid ‘© 2010 Times Publishing (Hong Kong) Limited Physics Suecess Key for HKDSEE 4 ogi etemnsneen iy (c) Essays Year | Paper | Question No.| Marks Content State and explain the designs of a transformer. Describe the . working principle of a transformer. Use a current balance 2000 3 16 | to investigate how the magnetic force depends on the Paper IB length of a current-carrying conductor in a magnetic field. Describe the working principle of a moving-coil meter. Explain the forces exerted on twa long straight current- ie carrying wires. Describe how to produce a uniform 2001 | paper mB 4 16 | magnetic field by using current-carrying conductor. Explain the e.m.f, induced in a rod when the rod moves in a uniform magnetic field AL Derive the variation of the induced exm.f in a coil rotating 2002 | paper} 7 | jmauniform magnetic field with time Describe and explain the design features ofa moving- AL coil galvanometer that gives a linear scale and discuss 2003 3(¢) fg Paper 1B two factors that determine the current sensitivity of the galvanometer, Give the meaning of magnetic flux density State the relation ] between magnetic flux and magnetic flux density. Explain the 2004 | sper IB 4 16 | meaning of Faraday's law. Explain the e.mJ. induced in a coil by using Lenz's law. Explain the back e.m.f. ina dc. motor and discuss the factors alfecting the back e.mn.l State Faraday’s law. Explain how eddy currents are formed. Describe the structure and explain the working AL 2005 4 11 | principle of an ideal transformer. Describe an experiment Paper IB to show the magnetic flux density at the centre of a circular coil is proportional to the current in the coil, at [Explain why a copper disc rotates with a bar magnet underneath +2006 Paper UB 3 Bay it. Explain how a d.c. motor works and why a motor is i sometimes connected! in series with a ‘starting! resistor iL Derive the magnetic force acting on a conductor in terms 2007 3b) ? of current I. Find the induced e.m.f, in a rod when the Paper IIB rod moves across a uniform magnetic field. Derive the Hall voltage Vj, built up across a slice of He conductor. Show how the number of charge carriers per 2008 4 16 | atom cam be estimated by measuring Vj. Describe an Paper UB experiment to show how the magnetic field in a solenoid depends on the number of turns per unit length. ‘© 2010 Times Publishing (Hong Kong) Limited Dhysies Success Key for HKDSEE 4 q4 Electricity and Magnetism Year | Paper | Question No. | Marks Content Explain the meaning of magnetic flux. State Len2's law. 5000 | 4 i 16 _ | Find the induced e.m., in a rod when the rod moves Paper 1B across a uniform magnetic field. Alternating current generated by rotating a coil in a magnetic field. Examination Trend (a) Describe a magnetic field pattern of a magnetic field by using field lines. (b) Distinguish between electric field lines and magnetic field lines. (© Examine the magnetic effect of electric current and find the corresponding magnetic field strength by using suitable formulae such as B = ae for long current-catrying straight wite and B= at jiitfor + long solenoid. Also, resolve related problems, (@) Examine the effect of induced force on a current-carrying conductor in a magnetic field and find the magnitude of the force by using the formula F = Bl¢ sin 8. (©) Describe the structure of a simple d.c. motor and explain its working principle by using the tuming efiect on a current-carrying coil in a magnetic field. (D) Examine the effect of induced force on a charge moving in a magnetic field and Bnd the magnitude of the force by using the formula F = BQv sin @. Hence, explain why the charge undergoes circular motion in the magnetic field (g) Distinguish between drift velocity and random speed of change carriers. Hence, derive the relation T= nAvQ to show how a current depends on drift velocity (h) Derive Hall voltage Vi = a and apply Hall voltage to find an unknown magnetic field. (Apply Lenz’ law to explain the effect of electromagnetic induction and apply Faraday’ law to find the induced emg. (@D_ Describe the structures of simple d.c. and acc. generators and explain their working principles by using Lenz’ law, (0) Explain the occurrence of eddy currents and discuss its practical uses, e.g. induction cooker, () Distinguish between direct currents and alternating currents, Define the rm.s, value of an a.c. as the steady d.c. which delivers the same power to a pute resistor as the a.c, (mm) Describe the structure ofa simple transformer ancl explain its working principle by using Lenz’ law. V,_N, (x) Examine methods for improving the efficiency ofa tansformer and apply {== to solve problems (0) Explain the working principle of high voltage transmission and discuss its advantages. © 2010 Times Publishing (Hong Kong) Limited -6- Physies Success Key for HKDSEE 4 Unit 4 Electromagnetism | + @tisetut Formulae= Useful term Formula 1, Magnetic field due to long straight wire p= fot 2ar (B= magnetic field, 4p = permeability of free space, Te current, r= shortest distance from a point to the wire) 2, Magnetic field inside long solenoid = pionl umber of turns, £ = length of the solenoid, ‘number of turns per unit length) 3. Force on a current-carrying wire in a magnetic field F=Ble sind (F = induced magnetic force, = length of the wire in the magnetic field, @ =angle between B field and current 1) (© 2010 Times Publishing (Hong Kong) Limited ste Physics Suseess Key for HKDSEE 4 or Electricity and Mogretisot yey 4. Force on a moving charge | The direction of the induced magnetic force should be determined by ina magnetic field Fleming’ left-hand rule, 5. General current flow current | equation T= nAvQ (n = number of charge carriers per m’, A= cross-sectional area, v = drift velocity, Q = charges of each charge carrier) 6. Hall voltage i ni (Vy = Hall voliage, t= thickness of the slice along the B field) 7. Magnetic flux: A = BA=BAcosO (@ = magnetic flux, A = area, = angle between the normal to the area and the magnetic field) '© 2010 Times Publishing (Hong Kong) Limited Physis Success Key for HKDSEE 4 Unit 4 esecromapuato 8. Faraday’ law A ee (= induced em.{,, Ad = change of magnetic flux linkage, ‘At = time interval) 9. Induced e.m.f, on a metal {rection of motion rod moving in a magnetic field Bel ino the page e=Béeysin@ (v = moving speed of the metal rod, = angle between the rod and its moving direction) 10. Ratio of secondary voltage to primary voltage in a transformer { ¥% primary coi secondary col with tums wih turns a 1% and LY, = Vp 9% (V, = voltage across the secondary coil, V, = voltage across the primary coil, 'N, = number of turns of the secondary coil, N, = number of turns of the primary coil, I, = current flowing through the secondary coil, 1, = current flowing through the primary coil, 1 efficiency of the transformer) 11. Root-mean-square value of | If Fl) = Fycos wt or Fysin er, then the rm.s. value of F is a sinusoidal function Physies Suecess Key for HKDSEE 4 ‘© 2010 Times Publishing (Hong Kong) Limited a= 4. electricity and Magnetism &fxamination scoring Techniques —————___. 1. Applying Right-hand Grip Rule to Determine Magnetic Field (@_ Magnetic field produced by a long current-carrying straight wire Fight-hand thumb potnts in | the direction of the current "oa cue fingers incest the cretion coneenticTngs ‘ofthe magnetic ld Magnetic field in the pattern of concentric rings centred at the wire is produced around a long current-carrying straight wire as shown in the above figure. (ii). Magnetic field produced by a current-carrying circular coil i “The above figure shows the pattern of the magnetic field generated by a current-carrying circular coil. The magnetic field lines go through the coil at right angles to the plane of the coil as shown. The figure below shows the side-view of the magnetic field through the plane of the coil of which the direction can be determined by using the right-hand grid rule as shown in the following figure. a curled fingers represent the 7 current direction, then the Px erected thumb indicates the . magnetic field direction {© 2010/Times Publishing (Hong Kong) Limited =10- Pysies Success Key for HKDSEE 4 Unit A Electromagnetism ‘ (iii) Magnetic field produced by a current-carrying solenoid ‘The above figure shows the pattern of the magnetic field generated by a current-carrying solenoid ‘The solenoid behaves like a bar magnet with poles at its two ends and the polarities of the poles can be determnined by using the right-hand grid rule as shown in the following figure. the thumb points to the rnorth-pole of the solenoid ‘curled fingors represent the direction ofthe current 2. Studying Magnetic Field Lines The pattern of a magnetic field can be represented by using magnetic field lines with the following properties: () Field lines form closed loops. They go out from the north magnetic pole to the south magnetic pole outside a magnet and then go from the south pole to the north pole inside the magnet as shown in the following figure, (ii) Field lines are continuous lines and never cross each other. (iii) Magnetic field strength is proportional to the density of the lines. (iv) The neutral point in'a magnetic feld is the point at which the magnetic field vanishes, {© 2010 Times Publishing (Hong Kong) Limited “ie Physics Suceess Key for HKDSEE 4 om lectriciy and Magnetism se (¥) The direction of the magnetic field at a point on a field line directs along the tangent to the field line, which means when a compass is placed at that point, its needle will direct along the tangent as shown in the following figure. “Tre needles of the compasses direct along the direction of the tangent to the field fine. — a ; en WD ‘neutral point 3. Applying Fleming's Left-hand Rule When a current-carrying wire is placed in a magnetic field and the current is not parallel to the field, the wire will experience an induced magnetic force which is perpendicular to the plane containing the B field and the current. The direction of the force can be determined by using the Fleming’ left-hand rule as shown in the following figure. F: induced magnetic force ‘B: magnetic field T-current © 2010 Times Publishing (Hong Kong) Limited 12 Physics Success Key for HKDSEE 4 Cnit 4 Electromagnets ‘ 4. Applying Fleming's Right-hand Rule When a conductor cuts a magnetic field, an e.m.f, is induced in the conductor, The direction of the induced e.m.f. (or the induced current) can be determined by using the Fleming’ right-hand rule as shown in the following figure direction of motion of the conductor 5. Applying Lenz's Law magnetic ld intially beting int the paper Fig, (a) Fig, (b) A circular coil is enclosed in a uniform magnetic field as shown in Fig. (a). Fig. (b) shows how the magnetic field B through the coil varies with time ¢, The magnetic field directs into the paper through the coil when Bis positive Li i) Fordsr< = a a B directing into the paper increases with time, According to Lenz law, an emf. is induced in the coil to produce magnetic field directing out of the paper in order to oppose the change. Hence, by using right-hand grip rule, it can be found that the induced current flows in the anticlockwise direction in the coil as shown in the fotlowing figure. {© 2010 Times Publishing (Hong Kong) Limited -13- Physics Success Key for HKDSEE 4 O44 stevetyoninanetin go 1x (/& «| 1 a x | co net (i) Bor 7 Stes B directing into the paper decreases with time. According to Lenz’ law, an e.m.f is induced in the coil to produce magnetic field directing into the paper in order to oppose the change. Hence, the induced current flows in the clockwise direction in the eo oo en i) For z si< > B directing out of the paper increases with time, According to Lenz’ law, an e.m.f. is induced in the coil to produce magnetic field directing into the paper in order to oppose the change, Hence, the Induced current flows in the clockwise direction in the coil fi 3r For > << Gy) For se< B directing out of the paper decreases with time. According to Lenz law, an e.m.f. is induced in the coil to produce magnetic field directing out of the paper in order to oppose the change. Hence, the induced current flows in the anticlockwise direction in the coil. ‘The following figure shows how the induced current (or the induced e.m.f.) in the coil varies with time ¢. Att=0, z and T, the rate of change of the magnetic field is maximum. So the magnitude of the induced current should also be maximum, Au z and z . the rate of change of the magnetic field is steady. So the induced current should be zero. {© 2010 Times Publishing (Hong Kong) Limited “14 Physies Success Key for HKDSEE 4 gtd stermameon ly 6. Distinguishing between r.m.s. and d.c. v ve Mo r ° t q —— Fig. (@) Fig. () Fig, (a) shows the time variation of an a.c. supply in one period of time. Fig. (b) shows the cortesponding time variation of the square of the a.c. supply in Fig. (a). The rm.s. value of the ac. supply is defined as Vans = 4 f where A is the shaded area bounded by the graph in Fig, (b) with the x-axis in one period of time Vems has the physical meaning that when it is applied actoss a resistor, the heating effect on the resistor would be the same as the resistor connected to a d.c. supply of voltage Vgc = Vins {©2010 Times Publishing (Hong Kong) Limited 15 Dhysies Success Key for HKDSEE 4 ELA certs and Magan XK > common mistakes —$$_____ noe) oe Three parallel long straight wires P, Q and R are fixed in space in such a way that the cross-section of the wires forms an isosceles right angled triangle as shown in the figure, The three wires carry currents of the same magnitude and all the currents are flowing into the paper. Indicate in the figure the direction of the net electrostatic interaction force acting on cra leis quite often that students mix up the direction of the magnetic field determined by the right-hand grip rule with the direction of the electrostatic interaction force between parallel wires, rag ‘The electrostatic interaction force between two parallel ‘wires is attractive if the currents in the two wires flow in the same direction, The following figure shows the two electrostatic attractive forces acting on Ai due to the currents in the other two wires. Fon {© 2010 Ties Publish Hong Kong) Limited -16- Physics Success Key for HKDSEE 4 Unit 4 Eksrenaontsn | iy XX, x x} p 1 x xf x x}x x} x xlx x! A rectangular wire frame PQRS moves with a uniform speed to the right across a region of uniform magnetic field directing into the paper as shown in the figure. What is the direction of the induced current flowing in wire PS and which point, P or S, is at a higher potential? The induced current flows from S to P and so.S is at a higher potential. The student simply thinks that current flows from higher potential to lower potential, so S is at a higher potential Comrect solution The induced current flows from S to P-and is at a higher potential rn Wire PS cuts the magnetic field, so the emf. is induced on wire PS which delivers a ‘current flowing in the clockwise direction. Therefore P is at a higher potential Do sot rail Lunvform magnetic — field directing Into the paper ve 9 metal rod The figure shows a metal rod PQ resting on a smooth metal rail placed in a uniform magnetic field directing into the paper. Describe the motion of the rod PQ when the switch is closed. The rod starts accelerating to tl right winen there is a current flowing inthe rod. The student only considers the factor of the induced magnetic force due to the effect of 2 current-carrying rod in @ magnetic fel. © 2010 Times Publishing (Hong Kong) Limited -- Physies Success Key for HKDSEE 4 RA Het ns arti The rod accelerates to the right at first and then the acceleration decreases with time to zero. The tod moves with constant velogity at equilibrium, re ‘An induced magnetic force exerts on the rod when there is a current flowing in the rod. By Fleming's left-hand rule, it can be determined that the force directs to the right. So the rod accelerates to the right at first: However, a back e.m.f. is induced in the rod once the rod starts moving in the magnetic field, This causes the current flowing in the rod decreases with ingreasing speed of the tod. As a result, the acceleration decreases with time to zero until there is no current flowing in the rod. 2m The above figure shows a solenoid of diameter 1 cm and length 20 cm. The solenoid has 10 000 tums and it carries a current of 2 A. A circular wire of radius 2 cm is wound around the solenoid as shown. If the current in the solenoid decreases uniformly to zero in 10 s, find the magnitude of the induced em. in the coil. (Given: permeability of free space = 4 x 10" H m™) “The mangnetis field strength generated by the s Nl 4M X10" X10000X2 t 02 According to Faraday’s law, the induced ¢.n 0426%"x(0.02)° “5 p= MOEA OECD soar rng The student uses the area of the coil to find the magnetic flux through the col. ‘© 2010 Times Publishing (Hong Kong) Limited -18- Physies Suecess Key for HKDSEE 4 ‘The magnetic field strength generated by the solenoid is pe MoNI _ 42x10” x10 000x2 é 02 According to Faraday's law, the induced e.m.f. in the coil is =0.126 T = Ad _ BA _ 0.126% (0.005) aot 9x10" V Analysis We should use the cross-sectional area of the solenoid to find the magnetic flux through the coll as the magnetic flux density Outside the solenoid is practically zero. 2mm 2A Bim Sem: ‘The above figure shows a slice of an aluminum plate with a current of 2 A passing through it. There are 10 conducting electrons per m’ of aluminum and the charge of an electron is 1.6 x 10°” C. The dimension of the slice is 5 em x 2 mmx 5 mm. A uniform magnetic field of 1.5 T is applied normally downwards to the plate and covers the whole surface area, A Hall voltage Vi is then set up across the slice. Which surface of the slice has a positive polarity of Vy? Also, find Vy. The surface facing outside with dimension 5 tm x 5 vane has the positive polarity of Vi. Br 15X2 n@t 10% x1.6%10 no x210" The student takes the thickness across Vj 28 the thickness ¢ in the formula. The surface facing outside with dimension 5 cm x 5 mm has the positive polarity of Vy, i 15x2 ror Vn= = a Sexe nQt 10” x1.6x10""x5x10 1 in the formula is the thickness of the slice ‘along the direction ofthe magnetic fel. ‘© 2010 Times Publishing (Hong Kong) Limited -19- Physics Suecess Key for HKDSEE 4 F « Sided Electricity and Magnelisre ty The following figure shows the power transmission of a bell system in a classroom. A signal generator produces an a.c. signal and is stepped up by transformer X before transmitting to a classroom through a long transmission cable, Assume all transformers are ideal. The turns ratio of transformer Yis 10: 1 and rated value of the loudspeaker is °6 V, 12 W’. Ifthe total resistance of the cable is 120 Q and the loudspeaker ‘works at its rated value, find the power loss in the cable transformer x transformer ¥ ‘classroom loudspeaker Hence, the power loss lu the calle The student uses the formula pat to Pun = = s0~w ————__| find the power loss inthe cable but mistakes I the primary voltage of the transformer as the voltage drop across the cable. Correct solution The input (primary) current of transformer Y is. 12 b= t0x6 For ideal transformer, Pi, = Pay. So the primary Hienee, the power loss in the eable is Current which is the current flowing in the cable can Pow = 0.22% 120=248W ——_ | P. be obtained as /, = “/7" and then the power loss in the cable is given by Pas =/,'R. (©2010 Times Publishing (Hong Kong) Limited -20 Physics Sueeess Key for HKDSEE 4 Page total e@ Revision Testy (25 items, 1 mark each) J 1, Which of the following properties is NOT J common to both electric and magnetic field lines? A. The lines repel each other, B, The lines never eross each other. C. The lines form closed loops. D. The densities of the lines are proportional to the strength ofthe fields gO 2. ‘Two parallel long straight wires carry equal J} currents / in the same direction as shown in the figure, Which of the following descriptions about the magnetic field produced by the currents at points P, Q and R is/are correct? (1). The magnetic field at P is directed out of the paper. 2) The magnetic field strength at Q is the greatest (3) The magnetic field at R is directed into the paper. A. (Donly B. (2)only €. (and (3) only D. )and (3) only Oo {© 2010 Times Publishing (Hong Kong) Limited nate gM 4 Electromagnetism re Let 4 cans ev A compass is passed through a solenoid slong ‘straight path as shown in the figure. In which of the following diagrams is the north pole of the compass needle pointing in the correct directions when the compass isa points P, Q and R respectively? The effect ofthe earths magueti field may be ignored. oe 0 © 6 .S mm OS a. © 2 Qo 7s 6&6 & Which of the following statements about the ‘magnetic field generated inside a long solenoid is/ are correct? (1). The magnetic field strength near the two ends of the solenoid is less than that inside the solenoid. (2) The magnetic field strength can be inereased by increasing the length of the solenoid, (3) The magnetic field strength can be inereased by inserting an iron core into the solenoid. A. (only B. @2)only €. (and (3) only D. (2)and (3) only Physies Suceess Key for HKDSEE 4 gon Electricity and Magnetism oe s. ) ee There is a current flowing in a long straght wire as shown in the figure. Which of the following figures best represents the magnetic field produced by the current-carrying wire? X : B field into the paper ‘+: B field out of the paper A B. XXX XK eee ee HK HH eee eee xx ERX eevee KX XX KX eee eee wee eel xxx RX XI eee eee XX XxX X sees RxKX see ee aes e D. XK XX KX coeeee cece ee xx xx x x! cece XX KKK X eeeene XX KX KX 6. A long cylindrical solenoid of diameter 40 mm J has 8 000 turns per metre. A current of 0.2 A flows in the solenoid, Find the magnetic field inside the solenoid. Given that y= 42 x 107 Hm, AL 60x107T B. 20x10°T ©. 5.7x10°T D. 20x10°T O ‘© 2010 Times Publishing (Hong Kong) Limited 4 J oS -22- Page total ‘When a long straight wire is connected to a dc. supply, a magnetic field of 0.1 T is measured at a distance rm from the wire, If another long, straight wire of the same resistivity and the same length but of double thickness is connected to the same d.c, supply, find the magnetic field at the same distance rm from the wire, A. 0,025 xT B. 0.05 eT C. 02uT D. 04uT | The figure shows a simple electromagnet. Which of the following can increase the strength of the electromagnet? colt softsron Lope 4 vv (1). Replacing the battery with a 12 V ac, power supply. (2) Reducing the resistance of the variable resistor, (3) Winding more turns around the soft-iron core. AL (I)only B. Q)only C. (and @) only D. @)and@) only Physics Suecess Key for HKDSEE 4 Page total % i, & : th zomm oa a ‘The above figure shows two fixed parallel long straight wires, P and Q, separated by a distance of 20 mm, Each wire carries a current of 2 A flowing into the paper. Find the magnetic field strength at | 49, point A which is equidistant to the two wires and | yy the shortest lines joining A to the two wires make an angle of 90°. (Given: permeability of free space = 4a x 107 Hm AL 20nT B. 28yT C. 40nT D. 56uT oO 10. Pp metal y ral wad uri magnetic = — field directing into the paper gmetad The figure shows a metal rod PQ resting on a metal rail placed in a uniform magnetic field directing into the paper. When the switch is closed, which of the following is the correct.

You might also like